Behavioural Issues
Behavioural Issues
Behavioural Issues
BEHAVIOURAL ISSUES
194 QUESTIONS
[1] Source: Publisher
In a responsibility accounting system, the
process in which a supervisor and a
subordinate jointly determine the
subordinate's goals and plans for achieving
these goals is
A. Top-down budgeting.
B. Bottom-up budgeting.
C. Management by objectives.
D. Management by exception.
[2] Source: CMA 1292 3-30
Richmond Enterprises is reviewing its
policies and procedures in an effort to
enhance goal congruence throughout the
organization. The processes that are most
likely to encourage this behavior are
A. Participatory budgeting, reciprocal
cost allocation, and
management-by-objective performance
evaluation.
B. Reciprocal cost allocation,
zero-base budgeting, and standard
costing.
C. Cost-based transfer pricing, imposed
budgeting, and activity-based costing.
D. Cost-based transfer pricing,
management-by-objective performance
evaluation, and participatory budgeting.
[3] Source: CMA 0691 3-24
Most firms use return on investment (ROI)
to evaluate the performance of investment
center managers. If top management wishes
division managers to use all assets without
regard to financing, the denominator in the
ROI calculation will be
A. Total assets available.
B. Total assets employed.
C. Working capital plus other assets.
D. Shareholders' equity.
[4] Source: CMA 0691 3-29
The selection of the denominator in the
return on investment (ROI) formula is
critical to the measure's effectiveness.
Which denominator is criticized because it
combines the effects of operating decisions
made at one level of the organization with
financing decisions made at another
organizational level?
A. Total assets employed.
B. Working capital.
C. Total assets available.
D. Shareholders' equity.
[5] Source: CMA 1291 3-9
A segment of an organization is referred to
as an investment center if it has
A. Authority to make decisions affecting
the major determinants of profit
including the power to choose its
markets and sources of supply.
B. Authority to make decisions affecting
the major determinants of profit
including the power to choose its
markets and sources of supply and
significant control over the amount of
invested capital.
C. Authority to make decisions over the
most significant costs of operations
including the power to choose the
sources of supply.
D. Authority to provide specialized
support to other units within the
organization.
[6] Source: CMA 1292 3-24
Managerial performance can be measured in
many different ways, including return on
investment (ROI) and residual income (RI).
A good reason for using RI instead of ROI is
that
A. RI can be computed without regard
to identifying an investment base.
B. Goal congruence is more likely to be
promoted by using RI.
C. RI is well understood and often used
in the financial press.
D. ROI does not take into consideration
both the investment turnover ratio and
return-on-sales percentage.
[7] Source: CMA 0694 3-27
The Stonebrook Company uses a
performance reporting system that reflects
the company's decentralization of decision
making. The departmental performance
reports show actual costs incurred during
the period against budgeted costs. Any
variances from the budget are assigned to
the individual department manager who
controls the costs. Stonebrook is using a
type of system called
A. Transfer-pricing accounting.
B. Flexible budgeting.
C. Responsibility accounting.
D. Activity-based budgeting.
[8] Source: CMA 0694 3-28
DigitalTech uses an accounting system that
D. Contribution accounting.
[9] Source: CMA 1294 3-22
If a manufacturing company uses
responsibility accounting, which one of the
following items is least likely to appear in a
performance report for a manager of an
assembly line?
A. Supervisory salaries.
B. Materials.
C. Repairs and maintenance.
D. Equipment depreciation.
[10] Source: Publisher
In evaluating an investment center, top
management should concentrate on
A. Dollar sales.
B. Net income.
C. Profit percentages.
D. Return on investment.
[11] Source: Publisher
The return on investment calculation
considers only the following components:
S = Sales
I = Investment
NI = Net income
Which of the following formulas best
describes the return on investment
calculation?
A. (I S) x (S NI) = I NI
B. (S I) x (NI S) = NI I
C. (I S) x (NI S) = (I x NI) x (S x S)
D. (S I) x (S NI) = (S x S) (I x NI)
A. The retail price.
[12] Source: Publisher
Which of the following will not improve
return on investment if other factors are
constant?
A. Decreasing expenses or assets.
B. Increasing selling prices.
B. $34
C. $46
D. $50
[24] Source: CIA 0593 IV-16
Which of the following is the most
significant disadvantage of a cost-based
transfer price?
B. $128,700
C. $130,000
D. $169,000
C. Incremental costs.
D. Costs over which they have
significant influence.
[32] Source: Publisher
In a responsibility accounting system, a
feedback report that focuses on the
difference between budgeted amounts and
actual amounts is an example of
A. Management by exception.
B. Assessing blame.
C. Granting rewards to successful
managers.
D. Ignoring other variables for which
the budgeted goals were met.
[33] Source: Publisher
The format for internal reports in a
responsibility accounting system is
prescribed by
C. 4
D. 20
[41] Source: Publisher
(Refers to Fact Pattern #1)
For Segment A, ROI is
A. 25%
B. 20.8%
D. 8.3%
????????? ????????? ????????? ??????
????
Net income
$5,000
--- $90,000
Sales
60,000 $750,000 $135,000 1,800,000
Investment
24,000 500,000 45,000
-Net income as % of sales
----Turnover of investment
----ROI
--20%
7.5%
Minimum ROI--dollars
---- $120,000
Minimum ROI--%
20%
6%
--Residual income
--0- $2,250
-[37] Source: Publisher
(Refers to Fact Pattern #1)
For Segment B, net income as a percentage
of sales is
A. 8%
B. 6.67%
A. $30,000
C. 4%
B. $4,800
D. 10%
C. $120,000
[38] Source: Publisher
(Refers to Fact Pattern #1)
For Segment C, net income as a percentage
of sales is
A. 5%
D. $12,000
[44] Source: Publisher
(Refers to Fact Pattern #1)
For Segment B, the minimum dollar ROI is
B. 6.67%
A. $30,000
C. 4%
B. $6,750
D. 20%
C. $4,800
D. $45,000
B. 1.5
A. $6,750
C. 2.5
B. $4,800
D. 4
C. $120,000
D. $9,000
A. 20%
Cost
Investment Profit
Centers Centers
Centers
??????? ?????????? ???????
A.
B. 6%
C. 15%
D. 10%
Yes
Yes
Yes
Yes
No
No
No
Yes
Yes
No
No
No
B.
C.
D.
A. 20%
B. 6%
C. 15%
D. 10%
[48] Source: Publisher
(Refers to Fact Pattern #1)
In Segment A, the residual income is
A. $200
C. An organization chart.
B. $12,000
D. Variances between actual and
budgeted controllable costs.
C. $(30,000)
D. $(60,000)
[49] Source: Publisher
(Refers to Fact Pattern #1)
In Segment D, the residual income is
A. $12,000.
B. $(30,000)
C. $(60,000)
Yes
Yes
Yes
Yes
No
Yes
No
Yes
No
No
No
No
B.
D. $9,000
C.
[50] Source: Publisher
Production levels are expected to increase
within a relevant range. A flexible budget is
used. What are the anticipated effects on the
following?
Fixed Costs
per Unit
???????????
A.
Variable Costs
per Unit
??????????????
Increase
Increase
Increase
No change
Decrease
Decrease
Decrease
No change
B.
C.
D.
D.
B. $68,400
C. $60,000
D. $44,400
[55] Source: Publisher
Suboptimization occurs when a management
decision benefits the
Company
Profit Investment
As a Whole Center
Center
?????????? ?????? ??????????
A.
Yes
Yes
Yes
Yes
No
No
No
No
No
Direct labor
180,000
Factory overhead
84,000
$ 50,400
General, selling, and
administrative
36,000
57,600
????????
????????
Total
$540,000
$108,000
========
========
During year 2, Ash produced 360,000
hammers, which were sold for $2.40 each.
Georgia's investment in Ash was $600,000
and $840,000 at January 1, year 1 and
January 1, year 2, respectively. Georgia
normally imputes interest on investments at
18% of average invested capital.
B.
C.
D.
No
Yes
Yes
B. 25.7%
C. 30%
D. 36%
[59] Source: Publisher
(Refers to Fact Pattern #2)
Assume that Ash's net operating income was
$72,000 and its average invested capital
was $720,000. Determine Ash's residual
income (loss) for the year ended December
31, year 2.
A. $(57,600)
B. $(79,200)
B. (480,000/720,000) x
(720,000/120,000) = ROI
C. (720,000/480,000) x
(120,000/720,000) = ROI
D. (480,000/720,000) x
(120,000/720,000) = ROI
[57] Source: Publisher
The following information relates to Cinder
Co.'s Northeast Division:
C. $72,000
D. $(36,000)
[60] Source: Publisher
(Refers to Fact Pattern #2)
How many hammers did Ash have to sell in
year 2 to break even?
A. 120,000
B. 110,000
Sales
$600,000
Variable costs
360,000
Traceable fixed costs
60,000
Average invested capital
120,000
Imputed interest rate
8%
Cinder's residual income was
A. $170,400
B. $180,000
C. 100,000
D. 90,000
[61] Source: Publisher
(Refers to Fact Pattern #2)
Ash's contribution margin for the year ended
December 31, year 2 was
C. $189,600
A. $350,000
D. $230,400
B. $324,000
C. $309,600
D. $216,000
[62] Source: Publisher
(Refers to Fact Pattern #2)
Based on Ash's year 2 financial information,
A. $648,000
A. Flexible budgeting.
B. $708,000
B. Human resource management.
C. $718,800
C. Responsibility accounting.
D. $720,000
D. Capital budgeting.
[63] Source: CIA 0594 III-44
A firm prepared a segmented income
statement that included the following data
for its suburban marketing segment:
Fixed costs controllable by the suburban
marketing segment manager
$150,000
Fixed suburban marketing costs
controllable by corporate management $250,000
Fixed manufacturing costs allocated to
the suburban marketing segment
$110,000
Variable manufacturing costs
$200,000
Variable selling costs
$100,000
Variable administrative costs
$130,000
Net sales
$950,000
The best measure of the economic
performance of the suburban marketing
segment is:
A. $370,000
B. $10,000
C. $520,000
D. $120,000
[64] Source: CMA 1291 3-10
A segment of an organization is referred to
as a service center if it has
B. Profit center.
D. Investment center.
C. Cost center.
A. Profit center.
C. Reciprocal allocation.
B. Investment center.
D. Transfer price accounting.
C. Contribution center.
D. Cost center.
A. Contribution accounting.
C. Cost-benefit accounting.
B. Cost-benefit accounting.
D. Program budgeting.
C. Flexible budgeting.
D. Responsibility accounting.
[71] Source: CMA 0695 3-28
In responsibility accounting, a center's
performance is measured by controllable
costs. Controllable costs are best described
as including
A. Direct material and direct labor,
only.
B. Only those costs that the manager can
influence in the current time period.
C. Only discretionary costs.
D. Those costs about which the manager
is knowledgeable and informed.
[72] Source: CMA 1295 3-5
Responsibility accounting defines an
operating center that is responsible for
revenue and costs as a(n)
A. Profit center.
B. Revenue center.
C. Division.
D. Operating unit.
[73] Source: CMA 1296 3-16
Rockford Manufacturing Corporation uses a
responsibility accounting system in its
operations. Which one of the following
items is least likely to appear in a
performance report for a manager of one of
Rockford's assembly lines?
A. Direct labor.
B. Materials.
C. Repairs and maintenance.
D. Depreciation on the manufacturing
facility.
[74] Source: CMA 0692 3-23
The WK Company uses a performance
reporting system that reflects the company's
decentralization of decision making. The
departmental performance report shows one
C. Working capital.
D. Shareholders' equity.
A. Supervisory salaries.
B. Materials.
Carolina Sanders
???????? ???????
A.
Accept
Reject
B.
Accept
Accept
Accept
C.
[88] Source: CMA 0694 3-29
D.
Reject
Reject
Sanders
???????
Accept
Reject
Reject
Accept
B.
C.
Accept
Accept
Reject
Reject
D.
B. Shareholders' equity.
C. Working capital plus other assets.
D. Total assets available.
[92] Source: CMA 0692 3-14
The most fundamental responsibility center
affected by the use of market-based transfer
prices is a(n)
A. Production center.
B. Investment center.
C. Cost center.
D. Profit center.
[93] Source: CMA 0694 3-30
An appropriate transfer price between two
divisions of The Stark Company can be
determined from the following data:
Fabricating Division
Market price of subassembly
$50
Variable cost of subassembly
$20
Excess capacity (in units)
1,000
Assembling Division
Number of units needed
900
What is the natural bargaining range for the
two divisions?
B. Incremental cost.
C. Budgeted cost with or without a
markup.
D. Market price.
[98] Source: CIA 1188 IV-23
The price that one division of a company
charges another division for goods or
services provided is called the
A. Market price.
B. Transfer price.
C. Outlay price.
D. Distress price.
[99] Source: Publisher
Motivation is
assembly line?
A. Supervisory salaries.
B. Materials.
A. 34.78%
B. 22.54%
D. Depreciation on equipment.
C. 19.79%
D. 16.67%
[106] Source: CMA 0697 3-24
Residual income is a better measure for
performance evaluation of an investment
center manager than return on investment
because
A. The problems associated with
measuring the asset base are eliminated.
A. Scenario 1.
B. $28,000
B. Scenario 2.
C. $31,500
C. Scenario 3.
D. $35,000
D. Scenario 4.
[108] Source: CMA 0691 3-26
If a manufacturing company uses
responsibility accounting, which one of the
following items is least likely to appear in a
performance report for a manager of an
Revenue
30,000
Plant and equipment
17,200
If the imputed interest charge is 15% and
Webb wants to achieve a residual income
target of $2,000,000, what will costs have
to be in order to achieve the target?
A. $9,000,000
B. $10,800,000
C. $25,150,000
A. $731,240
D. $25,690,000
B. $948,760
C. $1,668,760
D. $1,680,000
[112] Source: Publisher
What is the weighted-average cost of capital
(WACC) to be used in the economic value
added (EVA) calculation?
A. 8%
No
No
No
Yes
Yes
Yes
Yes
No
B.
B. 8.89%
C. 9%
C.
D. 10%
D.
[Fact Pattern #5]
Dzyubenko Co. reported these data at
year-end:
Pre-tax operating income $ 4,000,000
Current assets
4,000,000
Long-term assets
16,000,000
Current liabilities
2,000,000
Long-term liabilities
5,000,000
The long-term debt has an interest rate of
8%, and its fair value equaled its book
value at year-end. The fair value of the
equity capital is $2 million greater than its
book value. Dzyubenko's income tax rate is
25%, and its cost of equity capital is 10%.
[113] Source: Publisher
(Refers to Fact Pattern #5)
The EVA is
A. $1,380,000
B. $1,620,000
C. $1,830,000
D. $3,000,000
A. Just beyond what subordinates are
likely to reach.
[114] Source: CMA 0697 3-30
(Refers to Fact Pattern #5)
James Webb is the general manager of the
Industrial Product Division, and his
performance is measured using the residual
income method. Webb is reviewing the
following forecasted information for his
division for next year:
Amount
Category
(thousands)
???????????????????
???????????
Working capital
$ 1,800
A. Authority.
B. Responsibility.
C. Accountability.
D. Controllability.
A. Market share.
B. Delivery performance.
C. Customer satisfaction.
B. Current costs.
C. Controllable costs.
D. Indirect costs.
[131] Source: Publisher
Managers are most likely to accept
allocations of common costs based on
A. Cause and effect.
B. Ability to bear.
A. Current cost.
C. Fairness.
D. Benefits received.
C. Historical cost.
D. Present value.
[128] Source: Publisher
An organization's managerial
decision-making model for capital budgeting
is based on the net present value of
discounted cash flows. The same
organization's managerial performance
evaluation model is based on annual
divisional return on investment. Which of
the following is true?
A. Divisional managers are likely to
maximize the measures in the
decision-making model.
B. Divisional managers are likely to
maximize the measures in the
performance evaluation model.
C. The manager has an incentive to
accept a project with a positive net
present value that initially has a
negative effect on net income.
D. The use of models with different
criteria promotes goal congruence.
their culpability.
[135] Source: CIA 0594 III-50
Which of the following is not an advantage
of teamwork compared with work
performed by individuals?
A. Teams provide support to the team
members.
B. Teams make decisions that are more
easily accepted.
C. Teams provide a clear link between
effort and outcome.
C. Problem solving.
D. Compromise.
A. Ambiguous jurisdictions.
B. Competition for scarce resources.
C. Status differential.
D. Superordinate goals.
[138] Source: CIA 0594 II-9
Which of the following is not an example of
positive reinforcement of behavior?
A. Paying a bonus to employees who
had no absences for any four-week
period.
B. Giving written warnings after only
every other absence.
C. Assigning a mentor to each
employee.
D. Having a lottery every month where
10% of the employees with no absences
receive a $200 bonus.
C. III only.
D. I, II, and III.
A. Be more conservative.
B. Evaluate more complete information.
C. Generate more alternatives.
D. Be more time consuming.
[145] Source: CIA 1195 II-1
Which of the following statements about
group decision making is generally
considered false?
A. There is a lack of responsibility for
group decisions.
C. It is difficult to go in-depth on a
given topic.
D. Trainees do not easily retain what
they learn in the classroom.
[149] Source: CIA 1195 II-33
Which one of the following statements about
quality circles is false?
A. A quality circle is typically
comprised of a group of 8 to 10
subordinates and supervisors.
B. Part of the quality circle concept
includes teaching participants
communication skills, quality strategies,
and problem analysis techniques.
C. Quality circles meet on the company
C. Authoritative command.
D. Altering the human variable.
[164] Source: CIA 0596 II-27
A worker receives outstanding performance
evaluations from his first three supervisors.
He tries hard to please his fourth supervisor,
but the more he accomplishes, the more
critical his new supervisor becomes. The
new supervisor is generally considered a
marginal performer. Which of the following
is the most likely source of the problem?
B. $2,500.
C. $4,000.
D. $10,000.
B. $4,000.
C. $30,000.
D. $0.
B. Lower income.
C. Higher finished goods inventory.
of production levels.
B. Allocation of fixed production costs
is arbitrary.
C. Production costs are uncontrollable
and should not be charged to
production.
D. Period costs are not necessary to
produce a product.
B. Be greater.
C. Be equal.
D. Be substantially greater or less
depending upon external factors.
[178] Source: Publisher
Absorption costing inventories all
manufacturing overhead costs. Variable
costing expenses which of the following as
period costs?
B. $3,400 unfavorable.
C. $3,000 favorable.
D. $3,400 favorable.
[184] Source: Publisher
A company's flexible budget shows an
expected fixed cost of $100,000 for
straight-line depreciation when sales total
50,000 units. If sales total 52,000 units, and
the actual cost of depreciation is $103,000,
what will be the budget variance?
A. $1,000 favorable.
B. $3,000 favorable.
[189] Source: Publisher
C. $1,000 unfavorable.
D. $3,000 unfavorable.
[185] Source: CMA Samp Q3-12
To meet Zylon Corporation's overall
objectives, the Frame Division has just
initiated a program to increase sales by
improving the manufacturing quality of its
products. The most appropriate management
level to be responsible for this program is
the
A. Sales manager.
B. Production manager.
C. Division president.
D. Receiving and inspection manager.
[186] Source: Publisher
If a financial manager/management
accountant has a problem in identifying
unethical behavior or resolving an ethical
conflict, the first action (s)he should
normally take is to
A. Consult the board of directors.
B. Discuss the problem with his/her
immediate superior.
C. Notify the appropriate law
enforcement agency.
D. Resign from the company.
CMA PART 3A
BEHAVIOURAL ISSUES
ANSWERS
[1] Source: Publisher
Answer (A) is incorrect because a
top-down budget is generated by top
management and distributed to
(imposed on) lower-level managers.
Answer (B) is incorrect because a
bottom-up budget is generated by
lower-level management and
aggregated as it moves through the chain
of command.
Answer (C) is correct. The management
by objectives (MBO) approach is a
procedure in which a subordinate and a
supervisor agree on goals and the
methods of achieving them and develop
a plan in accordance with that
agreement. The subordinate is then
evaluated with reference to the plan at
the end of the plan period.
Answer (D) is incorrect because the
management by exception approach
uses measurable standards and
deviations therefrom to determine when
management action is needed.
[2] Source: CMA 1292 3-30
Answer (A) is incorrect because
reciprocal cost allocation does nothing
to enhance goal congruence.
equipment.
[10] Source: Publisher
Answer (A) is incorrect because dollar
sales do not give a measure of operating
performance based on resources
required.
Answer (B) is incorrect because net
income does not give a measure of
operating performance based on
resources required.
Answer (C) is incorrect because profit
percentages do not give a measure of
operating performance based on
resources required.
Answer (D) is correct. Each investment
center of a business should be evaluated
based upon return on investment to
judge operating performance. ROI is
comparable to calculations made both
within and without a particular
organization. Management may review
the investment opportunities available
within or without the firm. In essence,
net income is stated as a proportion of
investment capital (resources required).
[11] Source: Publisher
Answer (A) is incorrect because I NI
is the reciprocal of return on
investment.
Answer (B) is correct. The first term in
the formula for return on investment is
the number of times investment capital
has turned over through the sales
mechanism (S I). This amount is
multiplied by the net income expressed
as a percentage of sales (NI S) to give
the return on investment (NI I). The
basic formula for return on investment
is therefore
(S I) x (NI S) = NI I.
Answer (C) is incorrect because sales
must be divided by investment (S I) to
determine the number of times
investment capital has turned over,
which results in NI I as return on
investment.
Answer (D) is incorrect because net
income must be expressed as a
percentage of sales (NI S), which
results in NI I as return on investment.
environment.
[15] Source: Publisher
Answer (A) is incorrect because
motivation is the desire and the
commitment to achieve a specific goal.
for Segment D.
Answer (D) is incorrect because
$12,000 results from multiplying sales
(rather than investment) by the minimum
rate of return percentage.
[44] Source: Publisher
(540,000)
????????
$324,000
(108,000)
????????
Net operating income
$216,000
========
Therefore, the return on average
investment was 30% ($216,000
$720,000).
Answer (D) is incorrect because 36%
results from using year 1 year-end
invested capital instead of average
invested capital.
somewhat arbitrary.
[82] Source: CMA 1291 3-7
Answer (A) is incorrect because fixed
operating assets are controlled by the
division manager and contribute to
profits.
Answer (B) is correct. An evaluation of
an investment center is based upon the
return on the investment base. These
assets include plant and equipment,
inventories, and receivables. Most
likely, however, an asset, such as land,
that is being held by the division as a
site for a new plant would not be
included in the investment base because
it is not currently being used in
operations. Total assets in use rather
than total assets available is preferable
when the investment center has been
forced to carry idle assets.
Answer (C) is incorrect because
inventories are operating assets that
contribute to profits and are controlled
by the division manager.
Answer (D) is incorrect because the
level of accounts payable is an
operating decision that should be
considered in the evaluation of the
division manager.
[83] Source: CMA 1292 3-21
Answer (A) is incorrect because
increasing sales and expenses by the
same dollar amount will not change
income or ROI.
Answer (B) is incorrect because
decreasing revenues and expenses by
the same percentage will reduce income
and lower ROI.
Answer (C) is incorrect because
increasing investment and operating
expenses by the same dollar amount
will lower ROI. The higher investment
increases the denominator, and the
increased expenses reduce the
numerator.
Answer (D) is correct. ROI equals
income divided by invested capital. If a
company is already profitable,
increasing sales and expenses by the
same percentage will increase ROI. For
example, if a company has sales of
$100 and expenses of $80, its net
income is $20. Given invested capital
of $100, ROI is 20% ($20 $100). If
sales and expenses both increase 10%
to $110 and $88, respectively, net
income increases to $22. ROI will then
be 22% ($22 $100).
[84] Source: CMA 0693 3-12
Answer (A) is incorrect because
Sanders and Carolina would accept the
project.
investment.
Answer (C) is incorrect because
19.79% results from not subtracting
general and administrative expenses in
the calculation of before-tax profit.
Answer (D) is correct. An investment
center is responsible for revenues,
expenses, and invested capital. Given
average plant and equipment of $1,775
and average working capital of $625,
the net investment is $2,400. Before-tax
profit is $400 ($4,000 sales - $3,525
cost of goods sold - $75 general
expenses). If before-tax ROI equals
before-tax profit divided by net
investment, the answer is 16.67% ($400
$2,400).
[106] Source: CMA 0697 3-24
Answer (A) is incorrect because the
methods use the same asset base.
Answer (B) is correct. Residual income
is the excess of the amount of the ROI
over a targeted amount equal to an
imputed interest charge on invested
capital. The advantage of using residual
income rather than percentage ROI is
that the former emphasizes maximizing
a dollar amount instead of a percentage.
Managers of divisions with a high ROI
are encouraged to accept projects with
returns exceeding the cost of capital
even if those projects reduce the
department's ROI.
Answer (C) is incorrect because the
methods use the same asset base.
Answer (D) is incorrect because use of
the residual income method requires a
knowledge of the cost of capital; thus,
arguments about the implicit cost of
interest may escalate with use of the
residual income method.
[107] Source: Publisher
Answer (A) is correct. Residual income
is the excess of the amount of the ROI
over a targeted amount equal to an
imputed interest charge on invested
capital. If a manager has $19,000,000
of invested capital ($17,200,000 of
plant and equipment + $1,800,000 of
working capital), a 15% imputed
interest charge equals $2,850,000.
Adding $2,000,000 of residual income
to the imputed interest results in a target
profit of $4,850,000. This profit can be
achieved if costs are $25,150,000
($30,000,000 revenue - $4,850,000
profit).
Answer (B) is incorrect because
Scenario 2 requires maximum costs of
$26,220,000 to reach the target.
Answer (C) is incorrect because
Scenario 3 requires maximum costs of
$25,330,000 to reach the target.
method used.
Answer (A) is incorrect because, under
variable costing, all product costs are
variable.
Answer (B) is correct. Proponents of
variable costing maintain that fixed
overhead costs are more closely
correlated to capacity to produce than
to the production of individual units.
Thus, allocation of fixed production
costs is arbitrary.
Answer (C) is incorrect because
production costs (e.g., direct materials,
direct labor, and variable factory
overhead) are deemed to be
controllable.
Answer (D) is incorrect because period
costs are best attributable to time
intervals but nevertheless are necessary
to production.